73 votos

Integrales de $\sqrt{x+\sqrt{\phantom|\dots+\sqrt{x+1}}}$ en funciones elementales

Deje $f_n(x)$ ser definidos de forma recursiva como $$f_0(x)=1,\ \ \ f_{n+1}(x)=\sqrt{x+f_n(x)},\tag1$$ es decir, $f_n(x)$ contiene $n$ radicales y $n$ acontecimientos de $x$: $$f_1(x)=\sqrt{x+1},\ \ \ f_2(x)=\sqrt{x+\sqrt{x+1}},\ \ \ f_3(x)=\sqrt{x+\sqrt{x+\sqrt{x+1}}},\ \dots\tag2$$

Las funciones $f_0(x)$, $f_1(x)$ y $f_2(x)$ son integrables en funciones elementales, por ejemplo: $$\int\sqrt{x+\sqrt{x+1}}\,dx=\left(\frac{2\,x}3+\frac{\sqrt{x+1}}6-\frac14\right)\sqrt{x+\sqrt{x+1}}+\frac58\ln\left(2\,\sqrt{x+1}+2\,\sqrt{x+\sqrt{x+1}}+1\right).\tag3$$


Pregunta: ¿existe un entero $n>2$ tal que $f_n(x)$ es integrable en funciones elementales?


Actualización: La pregunta es reposteado en MathOverflow como fue sugerido por el moderador.

36voto

Robert Bryant Puntos 606

He publicado una respuesta a esta pregunta como una respuesta a la pregunta sobre MathOverflow. Ver a mi (segundo) respuesta en http://mathoverflow.net/questions/171733. Sin embargo, como Krokop señaló, sería mejor tener una copia aquí en Matemáticas.SE a la pregunta original. Gracias a Krokop para la copia de mi respuesta a las Matemáticas.SE.

Voy a demostrar que no existe ninguna primaria antiderivada de $f_n$ al $n>2$.

Suponga $n>2$ (NOTA: Esto es importante, porque el argumento de abajo no trabajo para $n\le2$; el lector puede disfrutar de encontrar donde se rompe), y deje $K_n = {\mathbb C}\bigl(x,f_n(x)\bigr)$ ser la escuela primaria, el diferencial de campo generadas por $x$$f_n(x)$. A continuación, $K_n$ es el campo de meromorphic funciones en la normalización $\hat C_n$ de la curva algebraica $C_n$ definido por el grado mínimo $y$-monic polinomio $P_n(x,y)$ que satisface $P_n\bigl(x,f_n(x)\bigr) \equiv 0$. Este grado mínimo es $2^n$; por ejemplo, $P_2(x,y) = (y^2-x)^2-x-1$$P_3(x,y) = \bigl((y^2-x)^2-x\bigr)^2-x-1$, etc.

Desde $P_{n+1}(x,y) = (P_n(x,y)+1)^2-x-1$$n\ge 1$$P_1(x,y)=y^2-x-1$, uno ve que, al aplicar el Criterio de Eisenstein a $P_n(x,y)$ considera como un elemento de $D[y]$ $D$ la integral de dominio ${\mathbb C}[x]$, $P_n(x,y)$ es irreductible para todos los $n\ge 1$. Por lo tanto, $\hat C_n$ está conectado.

Será importante en lo que sigue observar que $K_n$ tiene una involución $\iota$ que corrige $x$ y envía $f_n(x)$$-f_n(x)$; esto es debido a que $P_n(x,y)$ es incluso un polinomio en $y$. El campo fijo de $\iota$${\mathbb C}\bigl(x,\,f_n(x)^2\bigr)$, y el $(-1)$-subespacio propio de $\iota$${\mathbb C}\bigl(x,\,f_n(x)^2\bigr)f_n(x) = K_{n-1}{\cdot}f_n(x)$.

Ahora, la curva de $C_n\subset \mathbb{CP}^2$ tiene sólo un punto de la línea en el infinito, es decir,$[1,0,0]$, pero la normalización $\hat C_n$ $2^{n-1}$ puntos de mentir sobre este punto. Se puede parametrizar como sigue: en Primer lugar, establecer el convenio que $\sqrt{u}$ significa que la única analítica de la función en el complejo de $u$-plano menos negativo del eje y $0$ que satisface $\sqrt1 = 1$$\bigl(\sqrt{u}\bigr)^2 = u$. Deje $\epsilon = (\epsilon_1,\ldots,\epsilon_{n-1})$ ser cualquier secuencia con ${\epsilon_k}^2=1$ y considerar la secuencia de las funciones de $g^\epsilon_k(t)$ definido por los criterios $g^\epsilon_1(t) = \sqrt{1+t^2}$$g^\epsilon_{k+1}(t) = \sqrt{1+\epsilon_{n-k}t g^\epsilon_k(t)}$$1\le k < n$. Elegir, como uno puede, $\delta_n>0$ lo suficientemente pequeño para que, al $t$ es complejo y satisface $|t|<\delta_n$, todas las funciones de $g^\epsilon_k$ son analíticas al $|t|<\delta_n$. En particular, se encuentra una expansión $$ g^\epsilon_n(t) = 1+\tfrac12\epsilon_1\,t + \tfrac18(2\epsilon_1\epsilon_2-1)t^2 + O(t^3). $$

También, es fácil comprobar que el disco en $\mathbb{CP}^2$ definido por $$ [x,y,1] = [1,\ t g^\epsilon_n(t),\ t^2]\qquad\text{para}\quad |t|<\delta_n $$ es un nonsingular parametrización de una rama de $C_n$ en un barrio de el punto de $[1,0,0]$. En la normalización $\hat C_n$, esto es, a continuación, un local parametrización de una vecindad de un punto de $p_\epsilon\in \hat C_n$. Obviamente, esto describe la $2^{n-1}$ puntos distintos en $\hat C_n$.

Al $x$ $f_n$ son considerados como meromorphic funciones en $\hat C_n$, de ello se deduce que no hay un único local de coordenadas del gráfico de $t_\epsilon:D_\epsilon\to D(0,\delta_n)\subset \mathbb{C}$ de un disco abierto $D_\epsilon\subset \hat C_n$ $p_\epsilon$ tal que $t_\epsilon(p_\epsilon)=0$, y en el que uno ha fórmulas $$ x = \frac1{{t_\epsilon}^2} \quad\text{y}\quad f_n(x) = \frac{g^\epsilon_n(t_\epsilon)}{t_\epsilon} = \frac{1+\tfrac12\epsilon_1\ t_\epsilon +\tfrac18(2\epsilon_1\epsilon_2-1)\ {t_\epsilon}^2} {t_\epsilon} + O({t_\epsilon}^2). $$ En particular, se deduce que el $f_n(x)$, como una función de meromorphic en $\hat C_n$, ha polar divisor igual a la suma de los $p_\epsilon$ y, por tanto, tiene un grado $2^{n-1}$. Por supuesto, esto implica que el divisor de cero de a $f_n(x)$ $\hat C_n$ deben ser de grado $2^{n-1}$.

Tenga en cuenta que las funciones de $g^\epsilon_k$ satisfacer $g^{-\epsilon}_k(-t) = g^{\epsilon}_k(t)$ donde $-\epsilon = (-\epsilon_1,\ldots,-\epsilon_{n-1})$. Esto implica que $\iota(p_\epsilon) = p_{-\epsilon}$ y que $t_\epsilon\circ\iota = -t_{-\epsilon}$.

Ahora, el $2^{n-1}$ ceros de $f_n(x)$ $\hat C_n$ son distintos, porque ellos son los ceros del polinomio $q_n(x) = P_n(x,0) = (q_{n-1}+1)^2-x-1$, y el discriminante de $q_n$, siendo la resultante de $q_n$$q_n'$, es claramente un entero impar, y por lo tanto no es cero. Por lo tanto, $C_n$ es un ramificada de cubierta doble de $C_{n-1}$, ramificados exactamente donde $f_{n}$ tiene sus ceros. Esto induce a una ramificada cubierta $\pi_n:\hat C_n\to \hat C_{n-1}$ que es exactamente el cociente de $\hat C_n$ por la involución $\iota$ (cuyos puntos fijos donde se $f_n$ tiene su ceros). Desde entonces se ha la de Riemann-Hurwitz fórmula $$ \chi(\hat C_n) = 2\chi(\hat C_{n-1}) - B_n = 2\chi(\hat C_{n-1}) - 2^{n-1}, $$ y $\chi(\hat C_1) = \chi(\hat C_2) = 2$, la inducción da $\chi(\hat C_n) = (3{-}n)2^{n-1}$, so the genus of $\hat C_n$ is $(n{-}3) 2^{n-2} + 1$. (Esto en realidad no será necesario más adelante, pero es interesante.)

La única polos de $x$ $f_n(x)$ $\hat C_n$ son los puntos de $p_\epsilon$, y el cálculo utilizando las anteriores expansiones muestra que, en un barrio de $p_\epsilon$, se tiene una expansión de la forma $$ f_n(x)\,\mathrm{d} x - \mathrm{d}\left(f_n(x)\bigl(\tfrac12\ x + \tfrac16\ f_n(x)^2\bigr) \right) = \left(\frac{ (1-\epsilon_1\epsilon_2) } {4{t_\epsilon}^2} + O({t_\epsilon}^{-1})\right)\ \mathrm{d} t_\epsilon\ . $$ Por lo tanto, la meromorphic diferencial $\eta$ $\hat C_n$ definido por el lado izquierdo de esta ecuación, en el peor de los casos, el doble de los polos en los puntos de $p_\epsilon$ y ningún otro de los polos.

Ahora, por el Teorema de Liouville, $f_n$ tiene primaria antiderivada si y sólo si $f_n(x)\ \mathrm{d} x$ y, por lo tanto, la forma $\eta$ se pueden expresar como combinaciones lineales finitas exacta de los diferenciales y de registro exacto de los diferenciales. Por lo tanto, $f_n(x)$ tiene primaria antiderivada si y sólo si $\eta$ es expresable en la forma $$ \eta = \mathrm{d} h + \sum_{i=1}^m c_i\,\frac{\mathrm{d} g_i}{g_i} $$ para algunos $h,g_1,\cdots g_m\in K_n$ y algunas constantes $c_1,\ldots,c_m$. Supongamos que estos existen. Desde $\eta$, en el peor de los casos, el doble de los polos en el $p_\epsilon$ y no otros polos, se deduce que el $h$ debe tener, en el peor de los casos, simples postes en los puntos de $p_\epsilon$ y no otros polos; de hecho, $h$ está determinada únicamente hasta una constante aditiva, por su expansión en $p_\epsilon$ en términos de $t_\epsilon$ debe ser de la forma $$ h = \frac{\epsilon_1\epsilon_2-1}{4t_\epsilon} + O(1). $$
Por otra parte, debido a $\eta$ es impar con respecto a $\iota$, se deduce que el $h$ (después de la adición de un adecuado constante si es necesario) también debe ser impar con respecto a $\iota$. Esto implica, en particular, que $h$ se desvanece en cada uno de los ceros de $f_n$ (lo que, por el argumento anterior, son simples ceros). Esto implica que $h = r\,f_n$ algunos $r\in K_{n-1}$ que no tiene polos y satisface $r(p_\epsilon) = (\epsilon_1\epsilon_2-1)/4$ por cada $\epsilon$. Sin embargo, desde la $r$ no tiene polos y $\hat C_n$ está conectado, se deduce que el $r$ es constante. Por lo tanto, no puede tomar los dos valores distintos $0$$-1/2$, ya que la ecuación de $r(p_\epsilon) = (\epsilon_1\epsilon_2-1)/4$ implica.

Por lo tanto, el deseado $h$ no existe, y $f_n$ no puede ser integrado en la escuela primaria los términos de cualquier $n>2$.

14voto

fcop Puntos 2891

Creo que lo que hay es altamente no tienen la oportunidad de tener un número entero $n>2$ tal que $f_n(x)$ es integrable en funciones elementales.

Las razones son principalmente debido a los procesos de eliminación de los radicales.

Para$f_0(x)$$f_1(x)$ , son trivialmente integrable en funciones elementales.

Para $\int f_2(x)~dx$ ,

Deje $u=\sqrt{x+1}$ ,

A continuación, $x=u^2-1$

$dx=2u~du$

$\therefore\int\sqrt{x+\sqrt{x+1}}~dx$

$=\int2u\sqrt{u^2+u-1}~du$ , que se puede expresar en funciones elementales.

A partir de $\int f_3(x)~dx$ ,

Deje $u=\sqrt{x+1}$ ,

A continuación, $x=u^2-1$

$dx=2u~du$

$\therefore\int\sqrt{x+\sqrt{x+\sqrt{x+1}}}~dx$

$=\int2u\sqrt{u^2-1+\sqrt{u^2+u-1}}~du$

Introducir la sustitución de Euler:

Deje $v=u+\sqrt{u^2+u-1}$ ,

A continuación, $u=\dfrac{v^2+1}{2v+1}$

$du=\dfrac{2v(2v+1)-(v^2+1)2}{(2v+1)^2}dv=\dfrac{2v^2+2v-2}{(2v+1)^2}dv$

$\therefore\int2u\sqrt{u^2-1+\sqrt{u^2+u-1}}~du$

$=\int2\dfrac{v^2+1}{2v+1}\sqrt{\left(\dfrac{v^2+1}{2v+1}\right)^2-1+v-\dfrac{v^2+1}{2v+1}}\dfrac{2v^2+2v-2}{(2v+1)^2}dv$

$=\int4\dfrac{(v^2+1)(v^2+v-1)}{(2v+1)^3}\sqrt{\dfrac{(v^2+1)^2-(v^2+1)(2v+1)+(v-1)(2v+1)^2}{(2v+1)^2}}~dv$ , el cual es altamente no tienen la oportunidad de expresar en funciones elementales.

13voto

Eric Lee Puntos 136

Esta es sólo una parcial intento de responder a la pregunta. He leído esta descripción de los diferenciales de la teoría de Galois para averiguar acerca de esto. Voy a tratar de presentar el correspondiente bits de esas notas---esto no es original, y todos los errores son míos.

En primer lugar, definir qué se entiende por función primaria. Deje $\mathbb{C}(x,g_1,\ldots,g_n)$ ser el conjunto de todas las funciones racionales de $x$ $g_i$ donde $g_i(x)$ son funciones de la $x$. Primaria campo de $K=\mathbb{C}(x,g_1,\ldots,g_n)$ es un campo de las funciones que cada una de las $g_j$ es una exponencial, logaritmo o algebraica de la función de un elemento de $\mathbb{C}(x,g_1,\ldots,g_{j-1})$. Tan elemental campo es cerrado bajo la diferenciación.

Liouville del teorema dice que si $f$ es una función en $K$ que tiene una escuela primaria antiderivada, que antiderivada puede ser cualquier función arbitraria de elementos de $K$, pero debe ser igual a $$ \sum_i c_i \log g_i + h, $$ para algunas funciones $g_i$, $h$ en $K$, y las constantes de $c_i$.

Para las funciones de $f_n$, $f_n^2=x+f_{n-1}$, podemos elegir funciones elementales a decir $K=\mathbb{C}(x,f_1,\ldots,f_n)$ (es decir, todas las expresiones racionales implican $f_i$$x$), y deducir que si $\int f_n$ es elemental, debemos tener $$ f_n = \sum_i c_i \frac{g_i'}{g_i} + h', $$ donde$g_i$$h$$K$.

Mientras esto no parece mucho, no significa que uno (con rigor) sabe que uno no siempre tiene que mirar a funciones como $\sqrt{x+1}\log\sqrt{x+1}$ o $e^{\sqrt{x+1}}$ cuando la integración de $\sqrt{x+\sqrt{x+1}}$, si "elemental" funciones son como se definieron anteriormente. En cuanto a cómo usted puede encontrar esas funciones $g_i$, $h$ o refutar su existencia, no tengo idea.

Traté de integrar a $f_3$$f_4$, debido a la suposición de una forma general y de problemas para los coeficientes, pero no funcionó.

La actualización. (Este es un trabajo en progreso, así que tal vez las afirmaciones y expresiones que será algo de un misterio. La cuestión es ciertamente interesante.)

No es un algoritmo, el Risch-Hermite-Trager algoritmo (ver sistema de Álgebra por Geddes, Czapor, Labahn, y también la Integración de Funciones Algebraicas por Trager), para la integración arbitraria expresiones algebraicas en forma elemental o de demostrar su integrales no elementales. La función de $f_3(x)$ es tal vez integrable en forma elemental, con $$\int f_3 = \frac{-3+8x+2f_2+3 f_2 f_3/x}{12}f_3 + \int q_3, $$ donde $\int q_3$ es una suma de términos logarítmicos como en el teorema de Liouville (pero soy incapaz de integrar de manera explícita, incluso en el caso de $f_2$ es difícil para mí: le he preguntado a las preguntas relacionadas con el aquí y aquí, pero mi conocimiento de la geometría algebraica es poca). $$ q_3 = \left(\left(11 x^2+23 x^3+x^4-13 x^5-x^6+x^7+\left(-6 x-13 x^2-x^3+18 x^4+6 x^5-6 x^6\right) f_2+f_1 \left(-4 x-9 x^2-9 x^3+15 x^5+x^6-4 x^7+\left(8+17 x-5 x^3-2 x^4-23 x^5+13 x^6\right) f_2\right)\right) f_3\right)/\left(32 x^2 (1+x) \left(-1-x+x^2\right) \left(-1-x+x^2-2 x^3+x^4\right)\right). $$ Estoy seguro de por el algoritmo que es integrable en primaria términos, si y sólo si es posible escribir $$ \int q_3 = \sum c_i \log v_i, $$ donde $c_i$ son números que pertenecen al campo de la $\mathbb{Q}$ extendido por las raíces del polinomio $$\begin{gathered} R(Z) = \left(-288966001+75889061888 Z^2-15925264777216 Z^4+88562225643520 Z^6+439804651110400 Z^8\right)^2 \times\\ \left(-14508481-784902408192 Z^2-757665491845120 Z^4-103142060671792840704 Z^6+635008673359146778624 Z^8\right)^2 \times\\ \left(46732663382478474361-89569724082204845664256 Z^2+904900289526729208095571968 Z^4+1144371243549912082526161076224 Z^6+3200188454940133648592935688601600 Z^8+11158139832684515154655834451715555328 Z^{10}+9545270929062413812474597798367805308928 Z^{12}-124380415953981823700469862336514528116736 Z^{14}+50404501905167945967686052562499557916672 Z^{16}\right)^2. \end{reunieron}, $$ y $v_i$ tienen ciertas prescrito raíces y los polacos, pero no sé cómo calcular el $c_i$ $v_i$ explícitamente (en teoría es posible encontrar o establecer que no existen).

El punto es que no es un algoritmo determinista para averiguar el algebraicas parte de la integral de la $\int f_3$, y la integral puede ser evaluado, o demostrado ser no-primaria mediante la construcción de un cierto polinomio $R(Z)$, cuyas raíces forman una base de más de $\mathbb{Q}$ del campo para que los coeficientes de $c_i$ pertenecen. Si todas las raíces son cero (no-cero integrando que sólo tiene cero términos logarítmicos), la integral se sabe que no primarias. De lo contrario, la logarítmica plazo está dada por $$\sum_{\beta,R(\beta)=0} c_\beta \log v_\beta, $$ donde (aproximadamente) $v_\beta$ es una función de $(x,y)$ que tiene una raíz/polo dondequiera $q_3\,dx$ (visto como una función de $x$ $y$ donde $y$ es un multivalor algebraica de la función de $x$) tiene un polo con residuo $\beta$, y no otras raíces o polos en cualquier otro lugar. Esto es complicado por un cambio de variables $x=z^{-2}$, $\hat y=z y$ es necesario antes, y el hecho de que la curva de $(x,y)$ tiene puntos singulares que hacen que algunas cosas están mal definidos. Tales funciones $v_\beta$ (que son los generadores de principal específico ideales) puede no existir algunas otras razones, pero nada acerca de la función de $q_3$ impide su existencia.

También he calculado $R(Z)$$f_4$, y se ha distinto de cero raíces, lo que significa integrabilidad de $f_4$ en primaria términos, es equivalente a encontrar un conjunto de las funciones de $v_\beta$; no sé si existen o no.

2voto

Michael Steele Puntos 345

Si usted escoge $f_0 = 1, K_0 = \Bbb C(X)$$f_{n+1} = \sqrt{X+f_n}, K_{n+1} = K_n(f_{n+1})$, el primero de los tres campos tienen género $0$, por lo que la integración de $f_{n}$ es tan fácil como la integración de una fracción racional.

Por ejemplo, si la ponemos a $w = \frac {f_1-1}{f_2-1}$ $K_2 = \Bbb C(w)$ y tenemos fórmulas

$$ x = \frac{3^4-8w^3+10w^2-4w}{(w^2-1)^2}, f_1 = \frac{-2a^2+2w-1}{w^2-1}, f_2 = \frac{w^2-3w+1}{w^2-1} $$

A continuación, la integración de $f_2dx$ es la integración de $f_2(dx/dw) dw$, y se obtiene un resultado en términos de $w$$\log(w^2-1)$.

Sin embargo, $K_3 = K_2(\sqrt{x+f_2}) = K_2(v = \sqrt{4w^4-11w^3+10w^2-w-1})$ es obtenido por la que se adhiere a la raíz cuadrada de un elemento de grado $4$, lo $K_3$ es una curva elíptica (los siguientes campos' género a crecer de manera exponencial). Y desea integrar el $1$forma $f_3 dx$ en esta curva.

Si no me equivoco, los polos de la $1$-formulario se encuentran en la $4$ $x = \infty$ (o más bien $w^2 = 1$). Un cálculo muestra que los residuos se $+1/4$ alrededor de los puntos (en las coordenadas $(v,w)$) $A =(+5,-1), B = (+1,+1)$ y $-1/4$ alrededor de los puntos de $C = (-5,-1), D = (-1,+1)$. Lamentablemente, $[A]+[B]-[C]-[D] \neq 0$. Creo que este es un punto de infinita orden en la curva elíptica por lo que no hay función racional podemos enchufar en un $\log$ a cancelar los residuos de la $1$-forma.

Y si no me equivoco, una primitiva de $f_3dx$ aún no puede ser expresado usando logaritmos y funciones elípticas.

i-Ciencias.com

I-Ciencias es una comunidad de estudiantes y amantes de la ciencia en la que puedes resolver tus problemas y dudas.
Puedes consultar las preguntas de otros usuarios, hacer tus propias preguntas o resolver las de los demás.

Powered by:

X